Series and sequences Problem Help please.

In summary, the conversation is about a math problem involving a limit. The person is stuck on part b) and is seeking help. They have already solved part a) and have provided their solution for it. There is a discussion about the behavior of a limit and possible solutions for part b). The person thanks the others for their suggestions and is open to more suggestions.
  • #1
juanitotruan77
44
0

Homework Statement


Well I am stuck with this math problem and i was wondering if you could help me:
https://fbcdn-sphotos-b-a.akamaihd.net/hphotos-ak-xfp1/v/t1.0-9/10997707_10200231195780272_1526570237911134602_n.jpg?oh=8d10e10d9c41470b751c36f4330864f1&oe=557F93B0&__gda__=1434457045_a894f47029b86afda4081e42d0282078
I already solved a), but i stuck with b)

2. The attempt at a solution
This is the solution i came up with for a)
https://fbcdn-sphotos-h-a.akamaihd.net/hphotos-ak-xfp1/v/t1.0-9/14059_10200231203980477_4182850412301272757_n.jpg?oh=6182f6ae981b50b7a0642552a1f53f57&oe=554DBAB5&__gda__=1434983726_505d6a8220d0e1cc493cbdeffc88483e


Thanks in advance
 
Physics news on Phys.org
  • #2
juanitotruan77 said:

Homework Statement


Well I am stuck with this math problem and i was wondering if you could help me:
https://fbcdn-sphotos-b-a.akamaihd.net/hphotos-ak-xfp1/v/t1.0-9/10997707_10200231195780272_1526570237911134602_n.jpg?oh=8d10e10d9c41470b751c36f4330864f1&oe=557F93B0&__gda__=1434457045_a894f47029b86afda4081e42d0282078
I already solved a), but i stuck with b)

2. The attempt at a solution
This is the solution i came up with for a)
https://fbcdn-sphotos-h-a.akamaihd.net/hphotos-ak-xfp1/v/t1.0-9/14059_10200231203980477_4182850412301272757_n.jpg?oh=6182f6ae981b50b7a0642552a1f53f57&oe=554DBAB5&__gda__=1434983726_505d6a8220d0e1cc493cbdeffc88483e


Thanks in advance

Maybe I don't see the problem. But if ##r_n## is increasing up to a limit ##L<1##, then isn't ##\frac{1}{1-r_n} \le \frac{1}{1-L}##?
 
  • Like
Likes juanitotruan77
  • #3
Dick said:
Maybe I don't see the problem. But if ##r_n## is increasing up to a limit ##L<1##, then isn't ##\frac{1}{1-r_n} \le \frac{1}{1-L}##?
That might be it, budd. Thanks. But i'd like to hear more suggestions, just in case.
 

1. What are series and sequences?

A series is a sum of terms in a specific order, while a sequence is an ordered list of numbers. Both involve finding patterns within a set of numbers.

2. How do I solve a series or sequence problem?

To solve a series or sequence problem, you can use various mathematical techniques such as finding the nth term, using formulas, or identifying patterns. It is important to carefully read and understand the given problem before attempting to solve it.

3. What is the difference between an arithmetic and geometric series?

An arithmetic series has a constant difference between each term, while a geometric series has a constant ratio between each term. In other words, in an arithmetic series, the difference between each term is the same, while in a geometric series, the ratio between each term is the same.

4. Can I use a calculator to solve series and sequence problems?

Yes, you can use a calculator to solve series and sequence problems. However, it is important to understand the concepts and formulas behind the calculations in order to use the calculator effectively.

5. How can series and sequences be applied in real life?

Series and sequences are used in various fields such as finance, engineering, and computer science. They can be used to model and predict patterns in data, make financial projections, and create algorithms for computer programs.

Similar threads

  • Calculus and Beyond Homework Help
Replies
3
Views
1K
  • Calculus and Beyond Homework Help
Replies
2
Views
1K
  • Introductory Physics Homework Help
Replies
12
Views
1K
  • Introductory Physics Homework Help
Replies
4
Views
1K
  • General Discussion
Replies
6
Views
2K
  • Introductory Physics Homework Help
Replies
9
Views
3K
  • Introductory Physics Homework Help
Replies
9
Views
901
  • Introductory Physics Homework Help
Replies
2
Views
1K
  • Calculus
Replies
2
Views
1K
Replies
10
Views
1K
Back
Top